Combine Hamiltonians of two different bases

  • Thread starter TheDestroyer
  • Start date
  • Tags
    Bases
In summary, the problem at hand involves creating a Hilbert Space by combining the bases and Hamiltonians of two systems, | \psi \rangle and | \phi \rangle. The new Hamiltonian, which introduces interactions between the two systems, is represented as \hat{H}_\psi \otimes I + I \otimes \hat{H}_\phi + \hat{H}_{int}. This can be achieved through the tensor product of the two basis sets and their respective Hamiltonians. An example of this is shown with the matrices H_1 and H_2.
  • #1
TheDestroyer
402
1
I'm trying to solve a dynamical quantum mechanics problem related to the Cs atom, but I'm having trouble in the following, and I'm afraid I'm doing it wrong.

Say I have the matrix form of the Hamiltonian on a basis for a system [itex]| \psi \rangle[/itex] to be [itex]H_\psi[/itex], and another system with bases [itex]| \phi \rangle[/itex] with Hamiltonian [itex]H_\phi[/itex].

Now I would like to introduce interactions between the first and seconed systems, which will become (I suppose) [itex]| \psi \phi\rangle[/itex].

1) How do I combine those hamiltonians in matrix formalism before adding the new hamiltonian that introduces the interactions?

2) Say the first and second system are complete angular momenta basis in Zeeman eigen-states (so [itex]|F_1 m_{F_1} \rangle[/itex] and [itex]|F_2 m_{F_2}\rangle[/itex]). If the Wigner-D rotation matrix for the first Hamiltonian/basis is [itex]\mathcal{D}^{F_1}_{m_{F_1} m_{F_1}^\prime}[/itex], and for the second system is [itex]\mathcal{D}^{F_2}_{m_{F_2} m_{F_2}^\prime}[/itex]. How can I rotate each system and then combine them to introduce interactions between them?

Thank you for any efforts.
 
Physics news on Phys.org
  • #2
TheDestroyer said:
Say I have the matrix form of the Hamiltonian on a basis for a system [itex]| \psi \rangle[/itex] to be [itex]H_\psi[/itex], and another system with bases [itex]| \phi \rangle[/itex] with Hamiltonian [itex]H_\phi[/itex].

Now I would like to introduce interactions between the first and seconed systems, which will become (I suppose) [itex]| \psi \phi\rangle[/itex].

You suppose correct. For this situation we create a Hilbert Space by the tensor product of basis of the two spaces containing Hamiltonians [itex]\hat{H}_\psi[/itex] and [itex]\hat{H}_\phi[/itex](which are spanned by [itex]| \psi \rangle[/itex] and [itex]| \phi \rangle[/itex] respectively).

TheDestroyer said:
1) How do I combine those hamiltonians in matrix formalism before adding the new hamiltonian that introduces the interactions?

So the new Hilbert Space is now spanned by basis [itex]| \psi_i \phi_j\rangle[/itex] or [itex]| \psi_i\rangle \otimes |\phi_j\rangle[/itex] (sum over i and j). The Hamiltonian of the system now becomes [itex]\hat{H}_\psi \otimes I + I \otimes \hat{H}_\phi + \hat{H}_{int}[/itex]. If you are not aware of this algebra, I would recommend Quantum Computation and Quantum Information by Neilsen and Chaung, Chapter 2.
 
Last edited:
  • #3
Thank you so much for your answer. Let me though put an example to make this clear, because I got some result that I don't believe. Say I have the Hamiltonian matrices [itex]H_1[/itex] and [itex]H_2[/itex]

[tex]H_1=\left(
\begin{array}{cccc}
\text{s1} & 0 & 0 & 0 \\
0 & \text{s2} & 0 & 0 \\
0 & 0 & \text{s3} & 0 \\
0 & 0 & 0 & \text{s4} \\
\end{array}
\right)[/tex]

[tex]H_2=\left(
\begin{array}{ccc}
\text{t1} & 0 & 0 \\
0 & \text{t2} & 0 \\
0 & 0 & \text{t3} \\
\end{array}
\right)[/tex]

So now we have:

[tex]H_1 \otimes I_3 = \left(
\begin{array}{cccccccccccc}
\text{s1} & 0 & 0 & 0 & 0 & 0 & 0 & 0 & 0 & 0 & 0 & 0 \\
0 & \text{s1} & 0 & 0 & 0 & 0 & 0 & 0 & 0 & 0 & 0 & 0 \\
0 & 0 & \text{s1} & 0 & 0 & 0 & 0 & 0 & 0 & 0 & 0 & 0 \\
0 & 0 & 0 & \text{s2} & 0 & 0 & 0 & 0 & 0 & 0 & 0 & 0 \\
0 & 0 & 0 & 0 & \text{s2} & 0 & 0 & 0 & 0 & 0 & 0 & 0 \\
0 & 0 & 0 & 0 & 0 & \text{s2} & 0 & 0 & 0 & 0 & 0 & 0 \\
0 & 0 & 0 & 0 & 0 & 0 & \text{s3} & 0 & 0 & 0 & 0 & 0 \\
0 & 0 & 0 & 0 & 0 & 0 & 0 & \text{s3} & 0 & 0 & 0 & 0 \\
0 & 0 & 0 & 0 & 0 & 0 & 0 & 0 & \text{s3} & 0 & 0 & 0 \\
0 & 0 & 0 & 0 & 0 & 0 & 0 & 0 & 0 & \text{s4} & 0 & 0 \\
0 & 0 & 0 & 0 & 0 & 0 & 0 & 0 & 0 & 0 & \text{s4} & 0 \\
0 & 0 & 0 & 0 & 0 & 0 & 0 & 0 & 0 & 0 & 0 & \text{s4} \\
\end{array}
\right)[/tex]

and

[tex]H_2 \otimes I_3 = \left(
\begin{array}{cccccccccccc}
\text{t1} & 0 & 0 & 0 & 0 & 0 & 0 & 0 & 0 & 0 & 0 & 0 \\
0 & \text{t2} & 0 & 0 & 0 & 0 & 0 & 0 & 0 & 0 & 0 & 0 \\
0 & 0 & \text{t3} & 0 & 0 & 0 & 0 & 0 & 0 & 0 & 0 & 0 \\
0 & 0 & 0 & \text{t1} & 0 & 0 & 0 & 0 & 0 & 0 & 0 & 0 \\
0 & 0 & 0 & 0 & \text{t2} & 0 & 0 & 0 & 0 & 0 & 0 & 0 \\
0 & 0 & 0 & 0 & 0 & \text{t3} & 0 & 0 & 0 & 0 & 0 & 0 \\
0 & 0 & 0 & 0 & 0 & 0 & \text{t1} & 0 & 0 & 0 & 0 & 0 \\
0 & 0 & 0 & 0 & 0 & 0 & 0 & \text{t2} & 0 & 0 & 0 & 0 \\
0 & 0 & 0 & 0 & 0 & 0 & 0 & 0 & \text{t3} & 0 & 0 & 0 \\
0 & 0 & 0 & 0 & 0 & 0 & 0 & 0 & 0 & \text{t1} & 0 & 0 \\
0 & 0 & 0 & 0 & 0 & 0 & 0 & 0 & 0 & 0 & \text{t2} & 0 \\
0 & 0 & 0 & 0 & 0 & 0 & 0 & 0 & 0 & 0 & 0 & \text{t3} \\
\end{array}
\right)[/tex]

So now the new Hamiltonian is

[tex]H_1 \otimes I_3 + I_4 \otimes H_2 = \left(
\begin{array}{cccccccccccc}
\text{s1}+\text{t1} & 0 & 0 & 0 & 0 & 0 & 0 & 0 & 0 & 0 & 0 & 0 \\
0 & \text{s1}+\text{t2} & 0 & 0 & 0 & 0 & 0 & 0 & 0 & 0 & 0 & 0 \\
0 & 0 & \text{s1}+\text{t3} & 0 & 0 & 0 & 0 & 0 & 0 & 0 & 0 & 0 \\
0 & 0 & 0 & \text{s2}+\text{t1} & 0 & 0 & 0 & 0 & 0 & 0 & 0 & 0 \\
0 & 0 & 0 & 0 & \text{s2}+\text{t2} & 0 & 0 & 0 & 0 & 0 & 0 & 0 \\
0 & 0 & 0 & 0 & 0 & \text{s2}+\text{t3} & 0 & 0 & 0 & 0 & 0 & 0 \\
0 & 0 & 0 & 0 & 0 & 0 & \text{s3}+\text{t1} & 0 & 0 & 0 & 0 & 0 \\
0 & 0 & 0 & 0 & 0 & 0 & 0 & \text{s3}+\text{t2} & 0 & 0 & 0 & 0 \\
0 & 0 & 0 & 0 & 0 & 0 & 0 & 0 & \text{s3}+\text{t3} & 0 & 0 & 0 \\
0 & 0 & 0 & 0 & 0 & 0 & 0 & 0 & 0 & \text{s4}+\text{t1} & 0 & 0 \\
0 & 0 & 0 & 0 & 0 & 0 & 0 & 0 & 0 & 0 & \text{s4}+\text{t2} & 0 \\
0 & 0 & 0 & 0 & 0 & 0 & 0 & 0 & 0 & 0 & 0 & \text{s4}+\text{t3} \\
\end{array}
\right)[/tex]

Is that how it's done?
 
  • #4
You have done correctly with some typos.

[tex]H_1=\left(
\begin{array}{cccc}
\text{s1} & 0 & 0 & 0 \\
0 & \text{s2} & 0 & 0 \\
0 & 0 & \text{s3} & 0 \\
0 & 0 & 0 & \text{s4} \\
\end{array}
\right),[/tex]

and define identity operator on first space

[tex]I_1=\left(
\begin{array}{cccc}
1 & 0 & 0 & 0 \\
0 & 1 & 0 & 0 \\
0 & 0 & 1 & 0 \\
0 & 0 & 0 & 1 \\
\end{array}
\right).[/tex]
Similarly
[tex]H_2=\left(
\begin{array}{ccc}
\text{t1} & 0 & 0 \\
0 & \text{t2} & 0 \\
0 & 0 & \text{t3} \\
\end{array}
\right),[/tex]
and identity operator on second space
[tex]I_2=\left(
\begin{array}{ccc}
1 & 0 & 0 \\
0 & 1 & 0 \\
0 & 0 & 1 \\
\end{array}
\right).[/tex]

The total Hamiltonian now should be [itex]\hat{H}_1 \otimes I_2 + I_1 \otimes \hat{H}_2 + \hat{H}_{int}[/itex]. We can evaluate terms as follows

[tex]H_1 \otimes I_2 = \left(
\begin{array}{cccccccccccc}
\text{s1} & 0 & 0 & 0 & 0 & 0 & 0 & 0 & 0 & 0 & 0 & 0 \\
0 & \text{s1} & 0 & 0 & 0 & 0 & 0 & 0 & 0 & 0 & 0 & 0 \\
0 & 0 & \text{s1} & 0 & 0 & 0 & 0 & 0 & 0 & 0 & 0 & 0 \\
0 & 0 & 0 & \text{s2} & 0 & 0 & 0 & 0 & 0 & 0 & 0 & 0 \\
0 & 0 & 0 & 0 & \text{s2} & 0 & 0 & 0 & 0 & 0 & 0 & 0 \\
0 & 0 & 0 & 0 & 0 & \text{s2} & 0 & 0 & 0 & 0 & 0 & 0 \\
0 & 0 & 0 & 0 & 0 & 0 & \text{s3} & 0 & 0 & 0 & 0 & 0 \\
0 & 0 & 0 & 0 & 0 & 0 & 0 & \text{s3} & 0 & 0 & 0 & 0 \\
0 & 0 & 0 & 0 & 0 & 0 & 0 & 0 & \text{s3} & 0 & 0 & 0 \\
0 & 0 & 0 & 0 & 0 & 0 & 0 & 0 & 0 & \text{s4} & 0 & 0 \\
0 & 0 & 0 & 0 & 0 & 0 & 0 & 0 & 0 & 0 & \text{s4} & 0 \\
0 & 0 & 0 & 0 & 0 & 0 & 0 & 0 & 0 & 0 & 0 & \text{s4} \\
\end{array}
\right),[/tex]
as you have done correctly.
Second term is
[tex]I_1 \otimes H_2 = \left(
\begin{array}{cccccccccccc}
\text{t1} & 0 & 0 & 0 & 0 & 0 & 0 & 0 & 0 & 0 & 0 & 0 \\
0 & \text{t2} & 0 & 0 & 0 & 0 & 0 & 0 & 0 & 0 & 0 & 0 \\
0 & 0 & \text{t3} & 0 & 0 & 0 & 0 & 0 & 0 & 0 & 0 & 0 \\
0 & 0 & 0 & \text{t1} & 0 & 0 & 0 & 0 & 0 & 0 & 0 & 0 \\
0 & 0 & 0 & 0 & \text{t2} & 0 & 0 & 0 & 0 & 0 & 0 & 0 \\
0 & 0 & 0 & 0 & 0 & \text{t3} & 0 & 0 & 0 & 0 & 0 & 0 \\
0 & 0 & 0 & 0 & 0 & 0 & \text{t1} & 0 & 0 & 0 & 0 & 0 \\
0 & 0 & 0 & 0 & 0 & 0 & 0 & \text{t2} & 0 & 0 & 0 & 0 \\
0 & 0 & 0 & 0 & 0 & 0 & 0 & 0 & \text{t3} & 0 & 0 & 0 \\
0 & 0 & 0 & 0 & 0 & 0 & 0 & 0 & 0 & \text{t1} & 0 & 0 \\
0 & 0 & 0 & 0 & 0 & 0 & 0 & 0 & 0 & 0 & \text{t2} & 0 \\
0 & 0 & 0 & 0 & 0 & 0 & 0 & 0 & 0 & 0 & 0 & \text{t3} \\
\end{array}
\right).[/tex]
(you did correctly but it seems lot of typesetting made you overlook the order [itex]I_1 \otimes H_2[/itex] :)).
And you can add these matrices to get total Hamiltonian matrix (without interaction).
 
  • #5
Hmmmmmmm... Thank you so much. Though I thought this approach would solve my problem, but apparently it doesn't.

I have Hyperfine states with [itex]F=3,4[/itex] (where [itex]F=I+J[/itex] is the sum of the total angular momentum and the nuclear spin), and I want to separate them, do a rotation with Wigner-D matrices, and combine them back. Do you know how this could be done?
 
  • #6
TheDestroyer said:
I have Hyperfine states with [itex]F=3,4[/itex] (where [itex]F=I+J[/itex] is the sum of the total angular momentum and the nuclear spin), and I want to separate them, do a rotation with Wigner-D matrices, and combine them back. Do you know how this could be done?

I am sorry, I don't know what Wigner-D matrices are.
 
  • #7
Thank you, I appreciate your help so far. You could though (if you're interested) look at this further. Wigner-D matrices are simply the matrices that provide rotations in the angular momentum basis with Euler angles.

So you have a basis of [itex]F=3[/itex] and [itex]m_F=-3,...,3[/itex], the Wigner-D matrix will be [itex]D_{m_F m_F^\prime}^{F=3}[/itex], where [itex]m_F, m_F^\prime[/itex] will be the matrix elements, and we'll have [itex]7\times 7=49[/itex] matrix elements for this basis.

That's the idea simply, so now, with that given, I know how to rotate the basis F=3 or F=4, but I don't know how to rotate them both when they're combined. You see where my problem is?
 
  • #8
TheDestroyer said:
That's the idea simply, so now, with that given, I know how to rotate the basis F=3 or F=4, but I don't know how to rotate them both when they're combined. You see where my problem is?
Please give one demonstration. Say I need to rotate basis F=3. How will I proceed?
 
  • #9
Thank you for your time. Wigner D-Matrix for that case will be the following, starting from the eigen-vector [itex]|F=3;m_F=-3 \rangle[/itex] to [itex]|F=3;m_F=3 \rangle[/itex]

[tex]
D^{F=3}=\left(
\begin{array}{ccccccc}
e^{-3 i \alpha -3 i \gamma } \cos ^6\left(\frac{\beta }{2}\right) & -\sqrt{6} e^{-3 i \alpha -2 i \gamma } \cos ^5\left(\frac{\beta }{2}\right) \sin \left(\frac{\beta }{2}\right) & \sqrt{15} e^{-3 i \alpha -i \gamma } \cos ^4\left(\frac{\beta }{2}\right) \sin ^2\left(\frac{\beta }{2}\right) & -2 \sqrt{5} e^{-3 i \alpha } \cos ^3\left(\frac{\beta }{2}\right) \sin ^3\left(\frac{\beta }{2}\right) & \sqrt{15} e^{i \gamma -3 i \alpha } \cos ^2\left(\frac{\beta }{2}\right) \sin ^4\left(\frac{\beta }{2}\right) & -\sqrt{6} e^{2 i \gamma -3 i \alpha } \cos \left(\frac{\beta }{2}\right) \sin ^5\left(\frac{\beta }{2}\right) & e^{3 i \gamma -3 i \alpha } \sin ^6\left(\frac{\beta }{2}\right) \\
\sqrt{6} e^{-2 i \alpha -3 i \gamma } \cos ^5\left(\frac{\beta }{2}\right) \sin \left(\frac{\beta }{2}\right) & \frac{1}{2} e^{-2 i \alpha -2 i \gamma } \cos ^4\left(\frac{\beta }{2}\right) (6 \cos (\beta )-4) & -\frac{1}{2} \sqrt{\frac{5}{2}} e^{-2 i \alpha -i \gamma } \cos ^3\left(\frac{\beta }{2}\right) (6 \cos (\beta )-2) \sin \left(\frac{\beta }{2}\right) & \sqrt{30} e^{-2 i \alpha } \cos ^2\left(\frac{\beta }{2}\right) \cos (\beta ) \sin ^2\left(\frac{\beta }{2}\right) & -\frac{1}{2} \sqrt{\frac{5}{2}} e^{i \gamma -2 i \alpha } \cos \left(\frac{\beta }{2}\right) (6 \cos (\beta )+2) \sin ^3\left(\frac{\beta }{2}\right) & \frac{1}{2} e^{2 i \gamma -2 i \alpha } (6 \cos (\beta )+4) \sin ^4\left(\frac{\beta }{2}\right) & -\sqrt{6} e^{3 i \gamma -2 i \alpha } \cos \left(\frac{\beta }{2}\right) \sin ^5\left(\frac{\beta }{2}\right) \\
\sqrt{15} e^{-i \alpha -3 i \gamma } \cos ^4\left(\frac{\beta }{2}\right) \sin ^2\left(\frac{\beta }{2}\right) & \frac{1}{2} \sqrt{\frac{5}{2}} e^{-i \alpha -2 i \gamma } \cos ^3\left(\frac{\beta }{2}\right) (6 \cos (\beta )-2) \sin \left(\frac{\beta }{2}\right) & e^{-i \alpha -i \gamma } \cos ^2\left(\frac{\beta }{2}\right) \left(\frac{15}{4} (\cos (\beta )-1)^2+5 (\cos (\beta )-1)+1\right) & -\frac{2 e^{-i \alpha } \cos \left(\frac{\beta }{2}\right) \left(\frac{15}{4} (\cos (\beta )-1)^2+\frac{15}{2} (\cos (\beta )-1)+3\right) \sin \left(\frac{\beta }{2}\right)}{\sqrt{3}} & e^{i \gamma -i \alpha } \left(\frac{15}{4} (\cos (\beta )-1)^2+10 (\cos (\beta )-1)+6\right) \sin ^2\left(\frac{\beta }{2}\right) & -\frac{1}{2} \sqrt{\frac{5}{2}} e^{2 i \gamma -i \alpha } \cos \left(\frac{\beta }{2}\right) (6 \cos (\beta )+2) \sin ^3\left(\frac{\beta }{2}\right) & \sqrt{15} e^{3 i \gamma -i \alpha } \cos ^2\left(\frac{\beta }{2}\right) \sin ^4\left(\frac{\beta }{2}\right) \\
2 \sqrt{5} e^{-3 i \gamma } \cos ^3\left(\frac{\beta }{2}\right) \sin ^3\left(\frac{\beta }{2}\right) & \sqrt{30} e^{-2 i \gamma } \cos ^2\left(\frac{\beta }{2}\right) \cos (\beta ) \sin ^2\left(\frac{\beta }{2}\right) & \frac{2 e^{-i \gamma } \cos \left(\frac{\beta }{2}\right) \left(\frac{15}{4} (\cos (\beta )-1)^2+\frac{15}{2} (\cos (\beta )-1)+3\right) \sin \left(\frac{\beta }{2}\right)}{\sqrt{3}} & \frac{1}{2} \left(5 \cos ^3(\beta )-3 \cos (\beta )\right) & -\frac{2 e^{i \gamma } \cos \left(\frac{\beta }{2}\right) \left(\frac{15}{4} (\cos (\beta )-1)^2+\frac{15}{2} (\cos (\beta )-1)+3\right) \sin \left(\frac{\beta }{2}\right)}{\sqrt{3}} & \sqrt{30} e^{2 i \gamma } \cos ^2\left(\frac{\beta }{2}\right) \cos (\beta ) \sin ^2\left(\frac{\beta }{2}\right) & -2 \sqrt{5} e^{3 i \gamma } \cos ^3\left(\frac{\beta }{2}\right) \sin ^3\left(\frac{\beta }{2}\right) \\
\sqrt{15} e^{i \alpha -3 i \gamma } \cos ^2\left(\frac{\beta }{2}\right) \sin ^4\left(\frac{\beta }{2}\right) & \frac{1}{2} \sqrt{\frac{5}{2}} e^{i \alpha -2 i \gamma } \cos \left(\frac{\beta }{2}\right) (6 \cos (\beta )+2) \sin ^3\left(\frac{\beta }{2}\right) & e^{i \alpha -i \gamma } \left(\frac{15}{4} (\cos (\beta )-1)^2+10 (\cos (\beta )-1)+6\right) \sin ^2\left(\frac{\beta }{2}\right) & \frac{2 e^{i \alpha } \cos \left(\frac{\beta }{2}\right) \left(\frac{15}{4} (\cos (\beta )-1)^2+\frac{15}{2} (\cos (\beta )-1)+3\right) \sin \left(\frac{\beta }{2}\right)}{\sqrt{3}} & e^{i \alpha +i \gamma } \cos ^2\left(\frac{\beta }{2}\right) \left(\frac{15}{4} (\cos (\beta )-1)^2+5 (\cos (\beta )-1)+1\right) & -\frac{1}{2} \sqrt{\frac{5}{2}} e^{i \alpha +2 i \gamma } \cos ^3\left(\frac{\beta }{2}\right) (6 \cos (\beta )-2) \sin \left(\frac{\beta }{2}\right) & \sqrt{15} e^{i \alpha +3 i \gamma } \cos ^4\left(\frac{\beta }{2}\right) \sin ^2\left(\frac{\beta }{2}\right) \\
\sqrt{6} e^{2 i \alpha -3 i \gamma } \cos \left(\frac{\beta }{2}\right) \sin ^5\left(\frac{\beta }{2}\right) & \frac{1}{2} e^{2 i \alpha -2 i \gamma } (6 \cos (\beta )+4) \sin ^4\left(\frac{\beta }{2}\right) & \frac{1}{2} \sqrt{\frac{5}{2}} e^{2 i \alpha -i \gamma } \cos \left(\frac{\beta }{2}\right) (6 \cos (\beta )+2) \sin ^3\left(\frac{\beta }{2}\right) & \sqrt{30} e^{2 i \alpha } \cos ^2\left(\frac{\beta }{2}\right) \cos (\beta ) \sin ^2\left(\frac{\beta }{2}\right) & \frac{1}{2} \sqrt{\frac{5}{2}} e^{2 i \alpha +i \gamma } \cos ^3\left(\frac{\beta }{2}\right) (6 \cos (\beta )-2) \sin \left(\frac{\beta }{2}\right) & \frac{1}{2} e^{2 i \alpha +2 i \gamma } \cos ^4\left(\frac{\beta }{2}\right) (6 \cos (\beta )-4) & -\sqrt{6} e^{2 i \alpha +3 i \gamma } \cos ^5\left(\frac{\beta }{2}\right) \sin \left(\frac{\beta }{2}\right) \\
e^{3 i \alpha -3 i \gamma } \sin ^6\left(\frac{\beta }{2}\right) & \sqrt{6} e^{3 i \alpha -2 i \gamma } \cos \left(\frac{\beta }{2}\right) \sin ^5\left(\frac{\beta }{2}\right) & \sqrt{15} e^{3 i \alpha -i \gamma } \cos ^2\left(\frac{\beta }{2}\right) \sin ^4\left(\frac{\beta }{2}\right) & 2 \sqrt{5} e^{3 i \alpha } \cos ^3\left(\frac{\beta }{2}\right) \sin ^3\left(\frac{\beta }{2}\right) & \sqrt{15} e^{3 i \alpha +i \gamma } \cos ^4\left(\frac{\beta }{2}\right) \sin ^2\left(\frac{\beta }{2}\right) & \sqrt{6} e^{3 i \alpha +2 i \gamma } \cos ^5\left(\frac{\beta }{2}\right) \sin \left(\frac{\beta }{2}\right) & e^{3 i \alpha +3 i \gamma } \cos ^6\left(\frac{\beta }{2}\right) \\
\end{array}
\right)
[/tex]

where the angles are the Euler angles, and the matrix is a unitary matrix, so I can use it to transform any Hamiltonian in the F=3 basis as follows:

[tex]
H^\prime=D^\dagger H D
[/tex]

The calculation of that matrix is another story, I got that from Mathematica. So my question is: assume I know that matrix for F=3 and F=4, and I know the Hamiltonians for both F=3 and F=4. How can I calculate the combined Hamiltonian after applying an arbitrary (but equal for each) transformations?

In other words: Say I use the matrix to rotate the F=3 Hamiltonian and F=4 Hamiltonian by some angles (a,b,c). Now I have 2 new Hamiltonians for each case. How can I combine them both?
 
Last edited:
  • #10
The hamiltonian matrix is
[itex] \hat{H} = \hat{H}_1 \otimes I_2 + I_1 \otimes \hat{H}_2 + \hat{H}_{int}[/itex]. The rotation matrix in the new space should be defined as
[itex] D = D^{F=3} \otimes I_2 + I_1 \otimes D^{F=4} [/itex]. And now you can use your formula
[tex]
\hat{H}^\prime=D^\dagger \hat{H} D.
[/tex]
 
Last edited:
  • #11
If I do this product the result's side length will become 7*9=63 elements... while my combined system from F=3 and F=4 has only 7+9=16 elements, since F=3 has side-length 2*3+1=7 and F=4 has 2*4+1=9.

So here's the problem right now.
 
  • #12
TheDestroyer said:
If I do this product the result's side length will become 7*9=63 elements... while my combined system from F=3 and F=4 has only 7+9=16 elements.
This seems to be about the difference between the direct product and the direct sum. The dimension of the product is n*m while the dimension of the sum is n+m. You get the direct sum if you write the initial matrices on the diagonal of the resulting matrix and fill the off-diagonals with zeros.

If you combine two systems, you need the direct product. But what you want to do is to combine different possible states of a single system, don't you? There, the direct sum seems to be the right thing to me.

I've never used Wigner's matrix but I remember that I found Sakurai really insightful on it at the time. Maybe it helps you.

/edit: typo
 
Last edited:
  • #13
TheDestroyer said:
while my combined system from F=3 and F=4 has only 7+9=16 elements, since F=3 has side-length 2*3+1=7 and F=4 has 2*4+1=9.
Your combined system also has 69 elements. Consider the first term of Hamiltonian. H (7*7 matrix) of system 1 tensored with Identity of system 2 (9*9 matrix) giving you a total of 63*63 matrix. Similarly the second term is 63*63 matrix.
 
  • #14
Thank you for your answers, guys.

Like Kith said, I need to add components and not have the product. A 63*63 matrix is redundant, where it gives me, for example, a state [itex]| F=3,F=4,m_F=3,m_F=-4 \rangle[/itex], which is definitely wrong. What I'm doing is adding components... please consider re-assessing the situation.
 

Related to Combine Hamiltonians of two different bases

1. What are Hamiltonians in the context of quantum mechanics?

In quantum mechanics, Hamiltonians are mathematical operators that represent the total energy of a quantum system. They describe how the system will evolve over time.

2. What is the significance of combining Hamiltonians of two different bases?

Combining Hamiltonians of two different bases allows us to describe the behavior of a quantum system in terms of two different sets of quantum states. This can provide a more complete understanding of the system and may be necessary for certain calculations or experiments.

3. How do you mathematically combine Hamiltonians of two different bases?

The combined Hamiltonian is found by taking the tensor product of the two individual Hamiltonians. This means multiplying each element of one Hamiltonian by the entire other Hamiltonian, and vice versa.

4. What is the physical interpretation of combining Hamiltonians of two different bases?

Combining Hamiltonians of two different bases can be thought of as combining two different ways of looking at the same quantum system. It allows us to see how the system behaves in both bases simultaneously, and how the two bases are related to each other.

5. Can combining Hamiltonians of two different bases be applied to any quantum system?

Yes, combining Hamiltonians of two different bases is a general method that can be applied to any quantum system. However, the specific techniques used may vary depending on the system and the bases being combined.

Similar threads

  • Quantum Physics
Replies
1
Views
977
  • Quantum Physics
Replies
6
Views
1K
  • Quantum Physics
Replies
12
Views
1K
Replies
3
Views
1K
Replies
9
Views
4K
Replies
4
Views
811
Replies
4
Views
818
  • Quantum Physics
Replies
1
Views
1K
  • Quantum Physics
Replies
24
Views
3K
Replies
5
Views
1K
Back
Top